Pagina 1 di 1

3. Ci riprovo (spero più difficile)

Inviato: 30 dic 2015, 23:55
da Talete
Sia $f: [0,1]\rightarrow \mathbb{R}$ tale che $f\in\mathcal C^0 ([0,1])$ ma $f\not\in\mathcal C^1 ([0,1])$.

(a) Dimostrare che, fissato $\varepsilon>0$, esistono un $\lambda\in(0,1)$ e una funzione $g: [0,1]\rightarrow \mathbb{R}$ tale che $g\in\mathcal C^1 ([0,1])$ e:
\[\lambda\cdot\int_{0}^1 [g'(x)]^2dx+(1-\lambda)\cdot\int_{0}^1 [f(x)-g(x)]^2dx < \varepsilon.\]

(b) È vero che, fissati $\varepsilon>0$ e $\lambda\in(0,1)$, esiste una funzione $g: [0,1]\rightarrow \mathbb{R}$ tale che $g\in\mathcal C^1 ([0,1])$ e:
\[\lambda\cdot\int_{0}^1 [g'(x)]^2dx+(1-\lambda)\cdot\int_{0}^1 [f(x)-g(x)]^2dx < \varepsilon?\]

(c) È vero che, fissati $\varepsilon>0$ e una funzione $g: [0,1]\rightarrow \mathbb{R}$ tale che $g\in\mathcal C^1 ([0,1])$, esiste $\lambda\in(0,1)$ tale che:
\[\lambda\cdot\int_{0}^1 [g'(x)]^2dx+(1-\lambda)\cdot\int_{0}^1 [f(x)-g(x)]^2dx < \varepsilon?\]

Spero non sia troppo banale come l'altro...

Re: 3. Ci riprovo (spero più difficile)

Inviato: 04 gen 2016, 12:43
da darkcrystal
Per essere onesti, (a) e (c) mi sembrano un po' stupidi (per (a) si può prendere $g(x)=0$ e $\lambda$ sufficientemente vicino ad 1, e per (c) i controesempi si sprecano...), ma (b) può essere simpatico (anche se non è molto difficile nemmeno lui)!

Re: 3. Ci riprovo (spero più difficile)

Inviato: 04 gen 2016, 13:28
da Talete
Sì sì lo so infatti erano messi per sviare l'attenzione quelli ;) lo so che anche (b) è facile (cioè, l'ho risolto pure io!) però più di tanto difficili non riesco a crearli.

Re: 3. Ci riprovo (spero più difficile)

Inviato: 04 gen 2016, 14:16
da darkcrystal
Np, (b) è effettivamente un problema carino! In realtà il messaggio era principalmente un tentativo di riattivare questa discussione, visto che un testo così lungo può un po' spaventare...

Re: 3. Ci riprovo (spero più difficile)

Inviato: 04 gen 2016, 18:13
da dario2994
Forse mi sto perdendo qualcosa, ma mi pare che la risposta a b) sia no.
Scegliamo una $f$ che rispetti le richieste e tale che $f(x)=0$ se $x\le \frac13$ e $f(x)=1$ se $x\ge\frac23$.
Fissiamo inoltre $\lambda=\frac12$.
Scegliamo arbitrariamente $g\in \mathcal C^1([0,1])$. Sia $A=\min g$ e $B=\max g$.
Allora è facile ricavare le seguenti disuguaglianze:
[math]
[math]

Da queste due disuguaglianze è chiaro che
[math]
Ma, indifferentemente da come scegliamo i valori di $A$ e $B$, il valore di destra è maggiore di $\frac37$ e ciò mostra che $\varepsilon$ non può essere scelto piccolo a piacere.

O qualcosa mi sta sfuggendo oppure non vedo il senso del problema.

Re: 3. Ci riprovo (spero più difficile)

Inviato: 09 gen 2016, 16:13
da dario2994
Scrivo per aggiungere due cose:

Una soluzione alternativa, più elegante ma meno elementare, sfrutta qualche proprietà dello spazio di Sobolev $H^1$ (e mostra che l'unico caso in cui la tesi è vera è con $f$ costante (in realtà anche la soluzione precedente lo mostra, bisogna fare qualche attenzione in più)).
Se la tesi fosse vera allora avremmo $f_n\stackrel{L^2}{\to} f$ e $f'_n\stackrel{L^2}{\to} 0$, ma per la chiusura dell'operatore di derivata, questo implicherebbe che $f\in H^1$ e $f'=0$, perciò $f$ sarebbe costante.

Ma devo aspettare conferma di correttezza o no? Sto aspettando invano?

Re: 3. Ci riprovo (spero più difficile)

Inviato: 09 gen 2016, 17:05
da Talete
Mi ero scordato questo topic. La conferma di correttezza c'è, scusami. Tutto giusto, vai pure! ;)